GRE 2025 Model Paper Set 4 Question Paper with Solutions PDF is available for download. The overall test time is about 1 hour and 58 minutes. GRE has total 5 sections:
- Analytical Writing (One "Analyze an Issue" task, Alloted time 30 minutes)
- Verbal Reasoning (Two Sections, with 12 questions and 15 questions respectively)
- Quantitative Reasoning (Two Sections, with 12 questions and 15 questions respectively)
GRE 2025 Model Paper Set 4 Question Paper with Solutions PDF
| GRE 2025 Model Paper Set 4 Question Paper with Solutions PDF | Check Solutions |
Which of the following is an acceptable seating arrangement, with the driver listed first under "Front Seat" and the passengers in the back seat listed from one side to the other side?
If Mohsen sits in the front seat, which of the following can be true?
If Theo sits in the front seat, which of the following must be true?
If both persons sitting in the front seat speak Hebrew, then it must be true that
Which of the following must be true if Orlando is the driver?

Question 6:
Which of the following, if true about early 1990, would most help to explain the decrease, in 1990, of the percent of people commuting to jobs in downtown Allentia who do so via public transportation?
View Solution
Step 1: Understanding the Concept:
The question asks for a reason that would explain the decrease in the percentage of people using public transportation in 1990, as shown by the dip in both lines on the graph for that year. We need to find a cause that would make public transport less attractive for commuters from both downtown and the outer suburbs.
Step 2: Detailed Explanation:
Let's analyze the options:
(A) Increased fares: A substantial increase in fares would make using public transportation more expensive. This would be a strong disincentive for people to use it, likely causing a decrease in ridership from all areas. This aligns perfectly with the data shown in the graph.
(B) New trains and buses: Adding new vehicles would improve the service, likely making it more attractive and \textit{increasing the percentage of commuters. This contradicts the graph.
(C) Improved security: Better security would make passengers feel safer, which would encourage more people to use the service, leading to an \textit{increase in the percentage. This contradicts the graph.
(D) Increased frequency: More frequent service would make public transport more convenient, which would likely lead to an \textit{increase in its use. This contradicts the graph.
(E) Adding new routes: Expanding the service to more areas would make it accessible to more people, which would also be expected to \textit{increase its use. This contradicts the graph.
Step 3: Final Answer:
The only option that provides a logical reason for a \textit{decrease in public transport usage is the increase in fares. Therefore, option (A) is the correct answer.
Quick Tip: In data interpretation questions, look for causes that match the effect shown in the data. If the graph shows a negative trend (a decrease), the correct explanation must be a negative factor (like higher cost, worse service, etc.). Quickly eliminate options that describe positive changes.
Which of the following, if true about early 1992, could most contribute to an explanation of the change, between 1991 and 1992, in the percent of those who commute via public transportation from the outer suburbs of Allentia, as compared to the change for the other group of commuters?
A new and more aggressive form of the fungus that caused the Irish potato famine of the nineteenth century has recently arisen. However, since this new form of the fungus can be killed by increased application of currently used fungicides, it is unlikely that the fungus will cause widespread food shortages in countries that currently rely on potatoes for sustenance.
Which of the following, if true, most calls into question the conclusion in the argument above?
Which of the following can be the musicians scheduled to teach the master classes, in the order in which they will teach, from first to sixth?
If R is scheduled to teach the second class, which of the following could be scheduled to teach the third class?
Which of the following must be true about the schedule of master classes?
If pianists are scheduled to teach the fourth, fifth, and sixth classes, which of the following must be true?
Which of the following must be true about the schedule of the master classes?
If classes are scheduled so that the classes taught by pianists and the classes taught by violinists alternate with one another, which of the following can be true?
If a violinist is scheduled to teach the first class and another violinist is scheduled to teach the sixth class, which of the following can be true?
Which of the following CANNOT be true about the schedule of the master classes?
Which of the following can be the list of the six people in the order of their offices, from office 1 through office 6?
If T is assigned to office 6, then U must be assigned to office
If Q is assigned to office 2, then the person assigned to office 6 must be
If Q is assigned to office 1, which of the following CANNOT be true?
If U is assigned to office 3, then Q must be assigned to office
If S is assigned to office 2, which of the following can be true?
As government agencies, faced with budget difficulties, reduce their funding for scientific research, a greater amount of such research is being funded by private foundations. This shift means that research projects likely to produce controversial results will almost certainly comprise a smaller proportion of all funded research projects, since private foundations, concerned about their public image, tend to avoid controversy.
Which of the following is an assumption on which the argument depends?
Juries in criminal trials do not base verdicts on uncorroborated testimony given by any one witness. Rightly so, because it is usually prudent to be highly skeptical of unsubstantiated claims made by any one person. But then, to be consistent, juries should end an all-too-common practice: convicting defendants on the basis of an uncorroborated full confession.
Which of the following, if true, most strengthens the argument above?
Although spinach is rich in calcium, it also contains large amounts of oxalic acid, a substance that greatly impedes calcium absorption by the body. Therefore, other calcium-containing foods must be eaten either instead of or in addition to spinach if a person is to be sure of getting enough calcium.
Which of the following, if true, most seriously weakens the argument above?
\( \displaystyle \frac{1}{4} - \frac{1}{5} \) & \( \displaystyle \frac{1}{20} \)
{\( x - y - 3 = 0 \)
Column A & Column B
\( x \) & \( y \)
The average (arithmetic mean) of 3 numbers is 37.5.
Column A: The sum of the 3 numbers
Column B: 100
\( x > 0 \)
Column A & Column B
\( \displaystyle \frac{1{x} + 1 \) & \( \displaystyle \frac{1}{x+1} \)
Column A: The perimeter of a square with sides of length 5
Column B: The perimeter of a rectangle with length 10 and width 2
x is positive number and y is 30 percent of x
Column A: 25. percent of y
Column B: 55 percent fo x
\( r \) & \( v \)
{\( x < 0 \)
{{Column A & {{Column B
\( |x| \) & \( x \)
\(m\) and \(n\) are positive integers.
Column A & Column B
\( m+n \) & \( mn \)
A bicycle registration costs $2.250$2.250 in City X and $3.00$3.00 in City Y. At these rates, the cost of 4 registrations in City X is k percent of the cost of 3 registrations in City Y.
Column A: k
Column B: 90
\( \left(\frac{1}{x}\right)^2 \) & \( x^2 \)

Column A: The area of the triangular region
Column B: 6
An identification code read from left to right consists of 2 digits, a dash, 3 digits, a dash, and then 4 digits. Each digit can be any number from 0 through 9.
Column A: The number of different identification codes possible
Column B: 109
In a rectangular coordinate system, line \(k\) has \(x\)-intercept 4 and slope -2.
The \(y\)-intercept of \(k\) & 2
Of the following, which is the closest approximation to \( \displaystyle \frac{(1.5)(19.9)(4.012)}{3.02} \)?
If \( (x-1)^2 = (x-2)^2 \), then \(x=\)
Question 43:

In the figure above, the areas of square regions X and Y are 1 and 4, respectively. What is the area of the triangular region?
If erasers cost
(0.25 each, at most how many erasers can be purchased for n dollars, where n is an integer?
Three salespeople are paid commissions in proportion to the amount of their sales, which total $25,000, $40,000, and $60,000 respectively. If a total of $20,000 is allocated for these three commissions, what is the amount of the largest commission paid?
Question 46:

When compared with the transaction fee for a
(1,000 cash advance, the transaction fee for a
)500 cash advance is
For which of the following cash advance amounts is the transaction fee approximately
(4?
For a total of
(1,500 that is advanced in separate cash amounts, for which of the following is the total of the transaction fees the LEAST?
What is the median nighttime charge for 360 minutes of calling?
The daytime charge for 360 minutes of calling for phone service T is approximately what percent more than the nighttime charge?
View Solution
Step 1: Understanding the Concept:
This question asks for the percent increase from the nighttime charge to the daytime charge for phone service T.
Step 2: Key Formula or Approach:
The formula for percent increase is: \[ Percent Increase = \frac{New Value - Original Value}{Original Value} \times 100% \]
Here, the "New Value" is the daytime charge, and the "Original Value" is the nighttime charge.
Step 3: Detailed Explanation:
First, we need to read the charges for phone service T from the bar chart.
Daytime charge (light bar) for T:
(99.40
Nighttime charge (dark bar) for T:
)71.40
The "Original Value" (the value we are comparing to) is the nighttime charge,
(71.40.
The "New Value" is the daytime charge,
)99.40.
First, find the amount of the increase: \[ Increase =
(99.40 -
)71.40 =
(28.00 \]
Now, use the percent increase formula: \[ Percent Increase = \frac{
)28.00}{
(71.40} \times 100% \]
To approximate this calculation: \[ \frac{28}{71.4} \approx \frac{28}{70} = \frac{4 \times 7}{10 \times 7} = \frac{4}{10} = 0.4 \]
Converting the decimal to a percentage: \[ 0.4 \times 100% = 40% \]
Let's check with a slightly more precise calculation: \(28 / 71.4 \approx 0.392\). This is very close to 40%.
Step 4: Final Answer:
The daytime charge is approximately 40% more than the nighttime charge for service T.
Quick Tip: When calculating "percent more than," the number that comes after "than" is always the original value and goes in the denominator of the fraction. Approximating the numbers can make the division much easier.
Question 51:

A square dart board has four dark circular regions of radius 3 inches as shown in the design above. Each point on the dart board is equally likely to be hit by a dart that hits the board. What is the probability that a dart that hits the board will hit one of the circular regions?
If \(x\) increased by 50 percent is equal to 20, then \(x=\)
In the rectangular coordinate plane, point A has coordinates (-4, 0), point B has coordinates (0, 4), point C has coordinates (4, 0), and point D has coordinates (0, -4). What is the area of quadrilateral ABCD?
An experiment has three possible outcomes, I, J, and K. The probabilities of the outcomes are 0.25, 0.35, and 0.40, respectively. If the experiment is to be performed twice and the successive outcomes are independent, what is the probability that K will not be an outcome either time?
If the inside diameter of a cylindrical garden hose is 1 inch, what is the length, in inches, of a straight hose that can hold a maximum of 1 gallon of water? (1 gallon = 231 cubic inches)
It is assumed that scientists will avoid making ---- claims about the results of their experiments because of the likelihood that they will be exposed when other researchers cannot ---- their findings.
As long as the nuclear family is ---- a larger kinship group through contiguous residence on undivided land, the pressure to ---- and thus to get along with relatives is strong.
In contrast to the substantial muscular activity required for inhalation, exhalation is usually a ---- process.
The documentary film about high school life was so realistic and ---- that feelings of nostalgia flooded over the college-age audience.
Although Georgia O'Keeffe is best known for her affinity with the desert landscape, her paintings of urban subjects ---- her longtime residency in New York City.
Even though the survey was designated as an interdisciplinary course, it involved no real ---- of subject matter.
The failure of many psychotherapists to ---- the results of pioneering research could be due in part to the specialized nature of such findings: even ---- findings may not be useful.
EARPLUG: NOISE::
REVISE: MANUSCRIPT::
DAREDEVIL: AUDACITY::
CALCIUM: MINERAL::
DIRGE: GRIEF::
ABANDON: INHIBITION::
INAUGURATION: OFFICIAL::
SCORN: REJECT::
PROFLIGATE: SOLVENT::
With which of the following statements concerning the stiffening of aging tissues would the author most likely agree?
According to the passage, which of the following statements is true of the process that discolors and toughens foods?
According to the passage, which of the following is characteristic of enzymatic glycosylation of proteins?
According to the passage, which of the following statements is true of Amadori products in proteins?
Which of the following best describes the function of the third paragraph of the passage (lines 19-29)?
The passage suggests that which of the following would be LEAST important in determining whether nonenzymatic glycosylation is likely to have taken place in the proteins of a particular tissue?
If the hypothesis stated in lines 56-58 is true, it can be inferred that the crystallin proteins in the lenses of people with cataracts
The primary purpose of the passage is to
It can be inferred that the author of the passage regards Smith's argument as
The author of the passage implies that which of the following occurred after the Iroquois were resettled on reservations early in the nineteenth century?
Which of the following best expresses an opinion presented by the author of the passage?
DRONE:
CERTAINTY:
MORIBUND:
PROFANE:
PERSONABLE:
MIRE:
CONCEPTUAL:
SURFEIT:
TENACITY:
APPOSITE:
STYMIE:

x, y, and z are coordinates of three points on the number line above.
COLUMN A: xy
COLUMN B: xz
10 inches of snowfall is equivalent to 1 inch of rainfall. (1 foot = 12 inches)
COLUMN A: The number of inches of rainfall equivalent to 1 food of snowfall
COLUMN B: 1
b > 0
COLUMN A: a – b
COLUMN B: b – a
The geometric mean of any two positive numbers x and y is xyxy
COLUMN A: The geometric mean of 4 and 8
COLUMN B: The average arithmetic mean of 4 and 8
\begin{tabular}{p{5cm} p{5cm}}
\textbf{Column A} & \textbf{Column B}
\( \displaystyle \frac{16}{35} \) & \( \displaystyle \frac{4}{9} \)
1 < x < y
COLUMN A: x+4
COLUMN B: y
\begin{tabular}{p{5cm} p{5cm}}
\textbf{Column A} & \textbf{Column B}
The length of ST & The length of RS
}
\begin{tabular}{p{5cm} p{5cm}}
\multicolumn{2}{l}{\(x+5=3\)
\multicolumn{2{l{\(y=2x\)
Column A & Column B
\( x \) & \( y \)
1 gallon = 8 pints
1 quart = 2 pints
COLUMN A: The least number of half-pint bottles needed of to hold x quarts milk
COLUMN B: The least number of one-quart bottles needed to hold x gallons of milk

The figure shows a cube with edge of length e.
COLUMN A: The length of diagonal AB
COLUMN B: \( \sqrt{2} e \)
\begin{tabular}{p{5.5cm} p{5.5cm}}
\multicolumn{2}{l}{\( y = 3x - 1 \)
{Column A & {{Column B
\( x \) & \( \frac{y{3} + 3 \)
\begin{tabular}{p{5.5cm} p{5.5cm}}
&
\( 37 \times \frac{37}{36} \) & \( 37 + \frac{37}{36} \)
View Solution
Step 1: Understanding the Concept:
The problem asks us to compare two expressions involving the same numbers: \(37\) and \(\frac{37}{36}\). Specifically, we are asked to compare: \[ Column A: 37 \times \frac{37}{36}, \quad Column B: 37 + \frac{37}{36}. \]
At first glance, it might seem that a product and a sum would naturally differ, but we need to carefully compute both values to determine which is larger—or if they are equal.
Step 2: Key Formula or Approach:
We can approach this problem systematically using the following methods:
Express both Column A and Column B with a common denominator so that we can compare them directly.
Alternatively, use a general algebraic check: for two numbers \(a\) and \(b\), the product \(ab\) is equal to the sum \(a+b\) if \((a-1)(b-1) = 1\). This is derived as follows:
\[ ab = a+b \implies ab - a - b = 0 \implies (a-1)(b-1) = 1. \]
This formula can help confirm whether the two quantities are equal.
Step 3: Detailed Explanation:
Step 3.1: Evaluate Column A \[ Column A = 37 \times \frac{37}{36} = \frac{37 \times 37}{36} = \frac{1369}{36}. \]
This is a straightforward calculation of the product.
Step 3.2: Evaluate Column B \[ Column B = 37 + \frac{37}{36}. \]
To combine these terms, we write 37 as a fraction with denominator 36: \[ 37 = \frac{37 \times 36}{36} = \frac{1332}{36}. \]
Now add the second term: \[ Column B = \frac{1332}{36} + \frac{37}{36} = \frac{1332 + 37}{36} = \frac{1369}{36}. \]
Step 3.3: Compare Column A and Column B
Both Column A and Column B simplify to the same fraction: \[ \frac{1369}{36}. \]
Thus, the two quantities are exactly equal.
Step 3.4: Verification Using the General Formula
For two numbers \(a\) and \(b\), we can verify equality using: \[ ab = a+b \iff (a-1)(b-1) = 1. \]
Here, \(a = 37\) and \(b = \frac{37}{36}\): \[ a-1 = 37-1 = 36, \quad b-1 = \frac{37}{36} - 1 = \frac{1}{36}. \]
Multiply: \[ (a-1)(b-1) = 36 \cdot \frac{1}{36} = 1. \]
This confirms that indeed \(ab = a+b\), which agrees with our direct calculation.
Step 4: Observations and Conclusion
Even though one expression is a product and the other is a sum, in this specific case the two quantities turn out to be equal. This is due to the particular relationship between the numbers: one number is slightly greater than 1 and the other is large enough to satisfy \((a-1)(b-1) = 1\).
Step 5: Final Answer: \[ \boxed{Column A = Column B} \] Quick Tip: When comparing complex-looking arithmetic expressions, try to find an algebraic relationship. Factoring or finding a common denominator can reveal that the two expressions are actually the same, as in this case where \(a \times \frac{a}{a-1}\) is being compared to \(a + \frac{a}{a-1}\).
\begin{tabular}{p{5.5cm} p{5.5cm}}
&
0.01% of 1,000 & 1
}
View Solution
Step 1: Understanding the Concept:
This question requires calculating a percentage of a number and comparing it to another number. The key is to correctly convert the percentage to a decimal or fraction.
Step 2: Key Formula or Approach:
"P percent of a number" means \( \frac{P}{100} \times Number \).
In this case, P = 0.01.
Step 3: Detailed Explanation:
Column A: We need to calculate 0.01% of 1,000.
First, convert the percentage to a decimal. \[ 0.01% = \frac{0.01}{100} = 0.0001 \]
Now, multiply this decimal by 1,000. \[ 0.0001 \times 1,000 = 0.1 \]
Multiplying by 1,000 moves the decimal point 3 places to the right.
So, the quantity in Column A is 0.1.
Column B:
The quantity is 1.
Comparison:
We are comparing 0.1 (Column A) with 1 (Column B).
Since \(0.1 < 1\), the quantity in Column B is greater.
Step 4: Final Answer:
0.01% of 1,000 is 0.1, which is less than 1.
Quick Tip: Be very careful with decimal percentages. A common mistake is to confuse 0.01% with 0.01. Remember that the "%" sign means "divide by 100," so 0.01% is \(0.01 \div 100 = 0.0001\).
\begin{tabular}{p{5.5cm} p{5.5cm}}
\multicolumn{2}{l}{\(x\) and \(y\) are positive integers and \(x > y\).
{Column A & {{Column B
\( \displaystyle \frac{x^2{y^3} \) & \( \displaystyle \frac{y^3}{x^2} \)

\begin{tabular}{p{5.5cm} p{5.5cm}}
&
The perimeter of the shaded region in the rectangle & \( 2\sqrt{2} + 2 \)
\( -(\frac{-6}{2}) = \)
A rectangular parking lot 2x feet long and x feet wide is to be enlarged so that the lot will be 2 times as long and 3 times as wide as it is now. The area of the enlarged rectangular lot will be how many times the area of the present lot?
If \(2x = 5\) and \(3y = 8\), then \( \frac{4x}{9y} = \)
A certain jar contains 100 jelly beans: 50 white, 30 green, 10 yellow, 5 red, 4 purple, and 1 black. If a jelly bean is to be chosen at random, what is the probability that the jelly bean will be neither purple nor red?
The average (arithmetic mean) of \(|x|\) and \(x\) equals
Expenditures for physician services in 1989 were approximately how many billion dollars more than expenditures for nursing care?
View Solution
Step 1: Understanding the Concept:
This question asks us to find the difference in the dollar amount of expenditures between two categories shown in the pie chart. We need to use the percentages from the chart and the total expenditure amount.
Step 2: Key Formula or Approach:
1. Find the percentage for "Physician Services" and "Nursing Care" from the pie chart.
2. Calculate the difference between these two percentages.
3. Multiply this percentage difference by the total expenditure amount (
(550 billion) to find the difference in dollars.
Step 3: Detailed Explanation:
From the pie chart:
- Percentage for Physician Services = 20%
- Percentage for Nursing Care = 8%
First, find the difference in percentages: \[ Percentage Difference = 20% - 8% = 12% \]
Now, calculate what this percentage represents in terms of the total expenditure of
)550 billion. \[ Dollar Difference = 12% of
(550 billion \] \[ Dollar Difference = 0.12 \times 550 \]
To calculate this without a calculator: \[ 0.12 \times 550 = (0.10 \times 550) + (0.02 \times 550) \] \[ = 55 + (2 \times 5.5) = 55 + 11 = 66 \]
So, the difference is
)66 billion.
Alternatively, you could calculate each amount separately and then subtract:
- Physician Services expenditure = \( 20% \times 550 = 0.20 \times 550 =
(110 \) billion.
- Nursing Care expenditure = \( 8% \times 550 = 0.08 \times 550 =
)44 \) billion.
- Difference = \(
(110 -
)44 =
(66 \) billion.
Step 4: Final Answer:
The expenditures for physician services were
)66 billion more than for nursing care.
Quick Tip: When asked for the difference between two parts of a whole, it's often faster to find the difference in their percentages first and then calculate that percentage of the total. This saves you from performing two separate multiplication steps.
If health care expenditures accounted for 11 percent of the gross domestic product in 1989, then the gross domestic product was approximately how many billion dollars?
The total enrollment in 1985 was approximately how much greater than the total enrollment in 1960?
View Solution
Step 1: Understanding the Concept:
This question requires us to read data from a bar graph for two different years, calculate the total for each year, and then find the difference between these totals.
Step 2: Key Formula or Approach:
1. Read the male and female enrollment numbers for 1960 and 1985.
2. Calculate the total enrollment for each year by adding the male and female values.
3. Subtract the 1960 total from the 1985 total to find the difference.
Step 3: Detailed Explanation:
First, let's read the approximate values from the bar graph (in millions).
For 1960:
- Males (dark bar) \(\approx\) 2.2 million
- Females (light bar) \(\approx\) 1.3 million
- Total enrollment in 1960 \(\approx\) 2.2 + 1.3 = 3.5 million
For 1985:
- Males (dark bar) \(\approx\) 6.0 million
- Females (light bar) \(\approx\) 6.5 million
- Total enrollment in 1985 \(\approx\) 6.0 + 6.5 = 12.5 million
Now, find the difference: \[ Difference = Total in 1985 - Total in 1960 \] \[ Difference \approx 12.5 million - 3.5 million = 9.0 million \]
The total enrollment in 1985 was approximately 9 million greater than in 1960.
Step 4: Final Answer:
The difference in total enrollment is approximately 9 million.
Quick Tip: When reading from a bar graph, you often have to estimate the values. Read them as carefully as you can, but don't worry about extreme precision. The answer choices are usually spread out enough to accommodate small estimation errors.
For which of the years shown was the ratio of male to female enrollment greatest?
View Solution
Step 1: Understanding the Concept:
We need to find the year where the ratio of male enrollment to female enrollment was the highest. This means we are looking for the year where the number of males was largest in comparison to the number of females.
Step 2: Key Formula or Approach:
The ratio is \( \frac{Males{Females} \). We can either estimate this ratio for each year or visually inspect the graph. A larger ratio means the male bar is much taller than the female bar.
Step 3: Detailed Explanation:
Let's visually inspect the graph for each year. We are looking for the year where the dark bar (Males) is proportionally the largest compared to the light bar (Females).
- 1960: Male bar (\(\approx\) 2.2M) is significantly taller than the female bar (\(\approx\) 1.3M). The ratio is clearly greater than 1. \( \frac{2.2}{1.3} \approx 1.69 \)
- 1965: Male bar (\(\approx\) 3.5M) is taller than the female bar (\(\approx\) 2.2M). The ratio is greater than 1, but visually, the proportional difference seems smaller than in 1960. \( \frac{3.5}{2.2} \approx 1.59 \)
- 1970: Male bar (\(\approx\) 4.5M) is taller than the female bar (\(\approx\) 3.2M). The proportional difference continues to shrink. \( \frac{4.5}{3.2} \approx 1.41 \)
- 1975: Male bar (\(\approx\) 5.8M) is taller than the female bar (\(\approx\) 5.0M). They are getting very close in height. \( \frac{5.8}{5.0} = 1.16 \)
- 1980: The male bar (\(\approx\) 5.8M) and female bar (\(\approx\) 6.2M) are very close, and the female bar is now slightly taller. The ratio is less than 1.
- 1985: The female bar (\(\approx\) 6.5M) is clearly taller than the male bar (\(\approx\) 6.0M). The ratio is less than 1.
By both visual inspection and approximate calculation, the ratio of males to females was highest in 1960, when the male bar was proportionally much taller than the female bar. The ratio has consistently decreased over time.
Step 4: Final Answer:
The greatest ratio of male to female enrollment occurred in 1960.
Quick Tip: For ratio questions on bar graphs, you can often solve them by visual inspection. Look for the largest proportional difference between the two bars being compared. You don't always need to calculate the exact numbers if the visual difference is clear.
For which of the following periods was the percent increase in female enrollment the greatest?
If a certain town has 90 doctors and the ratio of male doctors to female doctors is 3 to 2, then the number of female doctors in this town is
Which of the following points (x, y) is NOT on the graph of \(y < 2x\)?
If apples sell for
(0.68 per pound and Juanita bought 36 apples for
)8.16, then the average (arithmetic mean) number of apples per pound was
The sum of the first 50 positive integers is 1,275. What is the sum of the integers from 51 to 100?
Question 20:

The figure above shows a semicircle with center O and a quarter circle with center R. If OQ = 4 and QR = 6, what is the ratio of the area of the shaded region to the area of the semicircular region?
Joyce: Three years ago the traffic commission modified our town's busiest intersection for better visibility, a commendable effort to cut down on traffic accidents there.
Gary: Over the past three years there have been more, not fewer, traffic accidents per week at that intersection, so the modification has increased the likelihood of accidents there
The answer to which of the following questions would be most useful in evaluating Gary's argument?
Women make up the majority of the population in the country, and many of the prescriptions written by doctors for tranquilizers are for women patients. The testing of these drugs for efficacy and the calibration of recommended doses, however, was done only on men. Not even the animals used to test toxicity were female.
The statements above, if true, best support which of the following as a conclusion?
Which of the following is an acceptable selection of species to be planted at the two locations?
If maple, spruce, and tupelo are planted at location 1, then which of the following must be the other species planted at location 1?
If maple, redbud, and walnut are planted at location 1, then any of the following species can be planted at location 2 EXCEPT
If linden and quince are planted at location 1, then which of the following must be true?
If each of the eight species is selected for planting and spruce is planted at location 2, then which of the following must also be planted at location 2?
If each of the eight species must be selected for planting and quince and tupelo are planted at location 1, then which of the following can be the other two species planted at location 1?
Gray wolves have been absent from a large national park for decades. Park officials wish to reestablish the wolves without jeopardizing any existing species of wildlife there. Since the park contains adequate prey for the wolves and since the wolves avoid close contact with people, reintroducing them would serve the officials' purpose without seriously jeopardizing visitors' safety.
Each of the following, if true, strengthens the argument above EXCEPT:
Osteoporosis is a disease that reduces bone mass, leading to fragile bones that break easily. Current treatments for osteoporosis such as estrogen or calcitonin help prevent further loss of bone but do not increase bone mass. Since fluoride is known to increase bone mass, administering fluoride to osteoporosis patients would therefore help make their bones less susceptible to breaking.
Which of the following, if true, most seriously weakens the argument above?
The closest distance from which an asteroid has been photographed using ground-based radar is 2.2 million miles, the distance from which the asteroid Toutatis was recently photographed. The closest photograph of an asteroid is of Gaspra, which was photographed from a distance of only 10,000 miles.
Which of the following can be properly concluded from the statements above?
Which of the following most logically completes the argument below?
Alone among living species, human beings experience adolescence, a period of accelerated physical growth prior to full maturity. Whether other hominid species, which are now all extinct and are known only through the fossil record, went through adolescence cannot be known, since
If the mouse is in chamber 2, and the other four animals are all in chambers, which of the following is a pair of chambers that must be empty?
If the mouse in chamber 2, the parakeet is in chamber 4, and all the other animals are in chambers, then the hen can be in chamber.
If no chamber contains more than one animal and each of the five animals is in a chamber, then there is a total of how many different chambers any one of which could be the chamber that contains the mouse?
If all five animals are in the chambers, the mouse is in chamber 2, and the frog's chamber is different from and not directly adjacent to the hen's chamber, then the parakeet must be in chamber
If the hen is in chamber 1 with another animal, one animal is in chamber 5, and two animals are in chamber 3, which of the following pairs of animals must be in chamber 3?
Which of the following is an acceptable assignment of students to groups?
If one of the groups of two consists of Luz and Mark, which of the following must be true?
If Nelson is assigned to the group to which Olga is assigned, which of the following can be true?
If Mark is assigned to a group to which neither John nor Pat is assigned, which of the following must be true?
If John and Olga are assigned to different groups from each other, which of the following CANNOT be true?
GRAND CITY CENSUS REPORTS (1950-1980)

Which of the following, if true, most helps to reconcile the increase in housing units with the decline in population shown in the table above?
Fossils of the coral Acrocora palmata that date from the last period when glaciers grew and consequently spread from the polar regions are found at ocean depths far greater than those at which A. palmata can now survive. Therefore, although the fossilized A. palmata appears indistinguishable from A. palmata now living, it must have differed in important respects to have been able to live in deep water.
The argument depends on the assumption that
Conservationists have believed that by concentrating their preservation efforts on habitats rich in an easily surveyed group of species, such as birds, they would thereby be preserving areas rich in overall species diversity. This belief rests on a view that a geographical area rich in one group of species will also be rich in the other groups characteristic of the entire regional climate zone.
Which of the following findings about widely scattered tracts 10 kilometers by 10 kilometers in a temperate climate zone would most seriously challenge the conservationists' assumptions?
In the nineteenth century, novelists and unsympathetic travelers portrayed the American West as a land of ---- adversity, whereas promoters and idealists created a ---- image of a land of infinite promise.
Honeybees tend to be more ---- than earth bees: the former, unlike the latter, search for food together and signal their individual findings to one another.
Joe spoke of superfluous and ---- matters with exactly the same degree of intensity, as though for him serious issues mattered neither more nor less than did ----.
The value of Davis' sociological research is compromised by his unscrupulous tendency to use materials ---- in order to substantiate his own claims, while ---- information that points to other possible conclusions.
Once Renaissance painters discovered how to ---- volume and depth, they were able to replace the medieval convention of symbolic, two-dimensional space with the more ---- illusion of actual space.
He had expected gratitude for his disclosure, but instead he encountered ---- bordering on hostility.
The diplomat, selected for her demonstrated patience and skill in conducting such delicate negotiations, ---- to make a decision during the talks because any sudden commitment at that time would have been ----.
CONDUCTOR: INSTRUMENTALIST::
QUARRY: ROCK
STICKLER: EXACTING::
WALK: AMBLE::
JAZZ: MUSIC::
REPATRIATE: EMIGRATION::
PLACEBO: INNOCUOUS::
DISSEMINATE: INFORMATION::
VOICE: QUAVER::
Which of the following best describes the author's attitude toward Gaskell's use of the method of documentary record in Mary Barton?
According to the passage, Mary Barton and the early novels of D. H. Lawrence share which of the following?
Which of the following is most closely analogous to Job Legh in Mary Barton, as that character is described in the passage?
It can be inferred from examples given in the last paragraph of the passage that which of the following was part of "the new and crushing experience of industrialism" (lines 46-47) for many members of the English working class in the nineteenth century?
It can be inferred that the author of the passage believes that Mary Barton might have been an even better novel if Gaskell had
Which of the following phrases could best be substituted for the phrase "this aspect of Mary Barton" in line 29 without changing the meaning of the passage as a whole?
The author of the passage describes Mary Barton as each of the following EXCEPT
The author of the passage is primarily concerned with
It can be inferred that one reason the fourteen models described in the passage failed to agree was that
It can be inferred that the primary purpose of the models included in the study discussed in the second paragraph of the passage was to
The information in the passage suggests that scientists would have to answer which of the following questions in order to predict the effect of clouds on the warming of the globe?
SUSPEND:
CREDULITY:
MILD:
IMPLEMENT:
DIFFIDENCE::
BYZANTINE:
PROCLIVITY:
PROTRACT:
VAUNTING:
HALE:
SEMINAL:





Comments